Where does the 4 come from in the derivative of an integral?

Click For Summary
The discussion revolves around finding the derivative of the integral u(1/r)((d/dr)[(r)(dV/dr)])=P. The user initially attempts to derive V but arrives at an incorrect result, missing the factor of 4 in the correct answer. Clarification is sought on the origin of the 4 in the expression V=(Pr^2)/4u+C(1)ln(r)+C(2). Participants emphasize the importance of rewriting the equation clearly and integrating correctly, leading to the conclusion that the constants u and P are indeed constant. The conversation highlights the need for careful formulation and understanding of differentiation and integration in this context.
juice34

Homework Statement


Can someone help me take the derivative of the integral
u(1/r)((d/dr)[(r)(dV/dr)])=P


Homework Equations





The Attempt at a Solution


my attempt yields V=(Pr^2)/(2u)+C(1), which is not right. The actual answer is V=(Pr^2)/4u+C(1)ln(r)+C(2). I am having trouble finding out where the 4 comes from could someone please explain to me what is going on. Thank YOU
 
Physics news on Phys.org
Is u a function of r? If not why write u(1/r) instead of just (u/r)? Are u and P constants? I think I know the answers to those questions but you should pose your question more carefully. Rewriting your equation using ' for d/dr:

(rV')' = Pr/u

Integrate:

rV' = Pr2/(2u) + C

Divide both sides by r and integrate again.
 
Last edited:
LCKurtz u and P are constant. And I am not sure i follow this part, (rV')' = r/u. Is it equal to d/dx(r(dV/dr))
 
Last edited by a moderator:
I accidently left off the P, which I just edited to correct.

(rV')' is d/dr (r dV/dr)

No x in there. Its much neater to write with primes.
 
LCKurtz i cannot thank you enough, you are a excellent help!
 
Question: A clock's minute hand has length 4 and its hour hand has length 3. What is the distance between the tips at the moment when it is increasing most rapidly?(Putnam Exam Question) Answer: Making assumption that both the hands moves at constant angular velocities, the answer is ## \sqrt{7} .## But don't you think this assumption is somewhat doubtful and wrong?

Similar threads

  • · Replies 10 ·
Replies
10
Views
1K
Replies
3
Views
2K
Replies
5
Views
2K
  • · Replies 9 ·
Replies
9
Views
2K
  • · Replies 3 ·
Replies
3
Views
1K
Replies
5
Views
2K
Replies
2
Views
1K
  • · Replies 6 ·
Replies
6
Views
2K
  • · Replies 7 ·
Replies
7
Views
2K
  • · Replies 14 ·
Replies
14
Views
2K